LSAT and Law School Admissions Forum

Get expert LSAT preparation and law school admissions advice from PowerScore Test Preparation.

 Administrator
PowerScore Staff
  • PowerScore Staff
  • Posts: 8919
  • Joined: Feb 02, 2011
|
#36499
Complete Question Explanation

Weaken. The correct answer choice is (A)


In this stimulus we are told that with regard to the treatment of certain illnesses, studies have shown
treatment X provides the same benefi ts as treatment Y. The author concludes that because treatment
X is less expensive and faster acting than treatment Y, but still provides the same patient benefi ts as
treatment Y, treatment X should be preferred to treatment Y for such illnesses.

he question stem asks which of the answer choices most weakens the argument contained in the
stimulus. It seems that treatment X is the better alternative. But what would weaken the conclusion
that treatment X should be preferred? The correct answer choice will convey some sort of inequality
that favors treatment Y; this will come in the form of either an added benefi t associated with
treatment Y or a previously unstated detriment associated with treatment X.

Answer choice (A): This is the correct answer choice. This is an example of an inequality between
treatments X and Y that favors treatment Y, by pointing to a new detriment associated with treatment
X, thereby weakening the conclusion that treatment X should be the preferred treatment.

Answer choice (B): The conclusion in the stimulus specifi cally refers to “these illnesses,” not other
illnesses. Because this answer choice does not deal with the referenced illnesses, it is incorrect and
should be eliminated.

Answer choice (C): The conclusion in the stimulus is concerned with the present, not the past. Once
upon a time, treatment Y may have been preferred on the basis of cost, but now it is more expensive
than treatment X. Since this is not a present inequality that favors treatment Y, this answer choice is
incorrect.

Answer choice (D): Just because a treatment is more popular does not mean that it should be the
preferred treatment, and this is merely a simple comparison of total prescriptions, inapplicable to
our limited discussion about particular illnesses. Because this answer choice does not present an
inequality that favors treatment Y, it does not weaken the argument in the stimulus.

Answer choice (E): If the stimulus stated that treatment X should be preferred to all other treatments,
this answer choice would weaken the conclusion. But because the conclusion in the stimulus only
concerns treatments X and Y, new information about treatment Z is irrelevant to the argument.
 jbeyer
  • Posts: 6
  • Joined: Jul 18, 2011
|
#2995
[LSAT Question Removed by Admin - December 2006 LR Section 2 question 1]

The correct answer is A. I don't understand why. The question stem does not refer to laboratory animals but to patients. Harmful side effects on laboratory animals might not have anything to do with side effects on humans. I feel like that wouldn't weaken the argument.

Thanks,

Jonathan
 Steve Stein
PowerScore Staff
  • PowerScore Staff
  • Posts: 1153
  • Joined: Apr 11, 2011
|
#2999
Thanks for your question. In that one, we are told that with regard to the treatment of certain illnesses, studies have shown treatment X provides the same benefits as treatment Y. The author concludes that because treatment X is less expensive and faster acting than treatment Y, but still provides the same patient benefits as treatment Y, treatment X should be preferred to treatment Y for such illnesses.

The question stem asks which of the answer choices most weakens the argument contained in the stimulus. It seems that treatment X is the better alternative. But what would weaken the conclusion that treatment X should be preferred? The correct answer choice will convey some sort of inequality that favors treatment Y; this will come in the form of either an added benefit associated with treatment Y or a previously unstated detriment associated with treatment X.

Answer choice A provides an example of an inequality between treatments X and Y that favors treatment Y, by pointing to a new detriment (or at least the newly presented potential for a new detriment) associated with treatment X, thereby weakening the conclusion that treatment X should be the preferred treatment.

Let me know whether that makes sense--thanks!
 jbeyer
  • Posts: 6
  • Joined: Jul 18, 2011
|
#3002
I understand the reasoning that you give that treatment X a detriment and then it becomes weaker. What I don't understand is how having side effects in lab animals a a detriment to patients. I would make the logical assumption that patients is referring to humans as no mention was made otherwise to the animalistic kingdom. Thus having side effects in lab animals has no bearing on how well X or Y would be to human patients.

If the question was talking about lab animals, or during testing to determine if the drug is good or not, then side effects in animals would make sense.

I just don't see the connection between lab animals and patients.

Thanks,

Jonathan
 Steve Stein
PowerScore Staff
  • PowerScore Staff
  • Posts: 1153
  • Joined: Apr 11, 2011
|
#3010
Hi Jonathan,

Thanks for your question. The conclusion of that argument is that X should be the preferred treatment. If, as answer choice A provides, that treatment produces harmful side effects in animals, this doesn’t guarantee that the treatment will hurt people, but all else equal, it certainly weakens the case for that treatment.

As evidence of this weakening power, consider the following example: you have a headache, and must choose between two medications, A and B. When you find out that they both cure headaches and A costs slightly less, it certainly sounds like A would be the preferred treatment. If I then mention that A is also known in most cases to cause horrible diseases in lab animals, that will most likely change your preference, by weakening the case for medication A.

Let me know if this clears that one up--thanks!
 tetsuya0129
  • Posts: 73
  • Joined: Jun 20, 2018
|
#80493
I got this one wrong for thinking laboratory animals not relevant. Then, after pondering quite a while, I came to realize why the answer choice is actually relevant: laboratory animal here is used as an object of a preposition to bring up a specific context, i.e. in a lab. Then, it is reasonable to say those animals' were for some research. So, if some research using animals showed treatment X less preferable than Y, it is reasonable to doubt whether treatment X is conclusively more ideal than Y.
User avatar
 KelseyWoods
PowerScore Staff
  • PowerScore Staff
  • Posts: 1079
  • Joined: Jun 26, 2013
|
#80726
Hi tetsuya!

Laboratory animals aren't irrelevant--there's a reason we use tests on laboratory animals before we implement treatments on humans. So, yes, if treatment X had negative side effects in lab tests but treatment Y did not, this would cast some doubt on whether or not X should be preferred over Y. Remember that we don't have to totally kill an argument in a Weaken question, just hurt it more than the other answer choices. Answer choice (A) is the only one that attacks the argument that X should be preferred over Y for these particular illnesses.

Hope this helps!

Best,
Kelsey

Get the most out of your LSAT Prep Plus subscription.

Analyze and track your performance with our Testing and Analytics Package.